Computing the volume of a simplex-like object with constraints












6












$begingroup$


For any $n geq 2$, let
$$D_n [r , (a_1, b_1 ) , ldots , (a_n, b_n) ] =
{ (x_1 , ldots , x_n ) in mathbb R^n mid
sum_i x_i = r mbox{ and } b_i geq x_i geq a_i , forall i },$$

where $r geq b_i geq a_i geq 0$ for all $i$, and all are real numbers.




Question: What is the 'volume' of $D_n [r , (a_1, b_1 ) , ldots , (a_n, b_n) ]$?




So for example for $n=2$, this set is either empty or it is some short line, and the purpose would be to calculate the length of that line (in terms of the parameters $r, a_i, b_i$). This is easy, I've done that already. Also the case $n=3$ would in principle still be doable to do by hand: it would be either zero (in case the set is empty) or part of a plane in $mathbb R^3$, the area of which we desire to compute.



Now to come up with a formula for the case $n=3$ (and higher) in a smarter way, my idea was to reason inductively from the case $n=2$, so basically reducing the three dimensional case to the two dimensional one, etc. I obviously tried to use integrals.



The problem I run into, is that in order to compute the volume we want with integrals, we have to view this set as (a subset of) an $n-1$-dimensional space. (Indeed, the integral of the constant function $1$ over the region $D_n [r , (a_1, b_1 ) , ldots , (a_n, b_n) ]$ seen as part of $mathbb R^n$ (rather than $mathbb R^{n-1}$), is equal to $0$. That's not what we want.) But to do that, it seems to me that we would need a concrete isometric embedding of $D_n [r , (a_1, b_1 ) , ldots , (a_n, b_n) ]$ into $mathbb R^{n-1}$, and I can't really find a nice one.



Do you have an idea about how to approach this problem best?



(This question was previously posted on Math.StackExchange, see https://math.stackexchange.com/questions/3135606/computing-hyper-area-of-a-contrained-simplex.)










share|cite|improve this question









New contributor




Oscar W. is a new contributor to this site. Take care in asking for clarification, commenting, and answering.
Check out our Code of Conduct.







$endgroup$

















    6












    $begingroup$


    For any $n geq 2$, let
    $$D_n [r , (a_1, b_1 ) , ldots , (a_n, b_n) ] =
    { (x_1 , ldots , x_n ) in mathbb R^n mid
    sum_i x_i = r mbox{ and } b_i geq x_i geq a_i , forall i },$$

    where $r geq b_i geq a_i geq 0$ for all $i$, and all are real numbers.




    Question: What is the 'volume' of $D_n [r , (a_1, b_1 ) , ldots , (a_n, b_n) ]$?




    So for example for $n=2$, this set is either empty or it is some short line, and the purpose would be to calculate the length of that line (in terms of the parameters $r, a_i, b_i$). This is easy, I've done that already. Also the case $n=3$ would in principle still be doable to do by hand: it would be either zero (in case the set is empty) or part of a plane in $mathbb R^3$, the area of which we desire to compute.



    Now to come up with a formula for the case $n=3$ (and higher) in a smarter way, my idea was to reason inductively from the case $n=2$, so basically reducing the three dimensional case to the two dimensional one, etc. I obviously tried to use integrals.



    The problem I run into, is that in order to compute the volume we want with integrals, we have to view this set as (a subset of) an $n-1$-dimensional space. (Indeed, the integral of the constant function $1$ over the region $D_n [r , (a_1, b_1 ) , ldots , (a_n, b_n) ]$ seen as part of $mathbb R^n$ (rather than $mathbb R^{n-1}$), is equal to $0$. That's not what we want.) But to do that, it seems to me that we would need a concrete isometric embedding of $D_n [r , (a_1, b_1 ) , ldots , (a_n, b_n) ]$ into $mathbb R^{n-1}$, and I can't really find a nice one.



    Do you have an idea about how to approach this problem best?



    (This question was previously posted on Math.StackExchange, see https://math.stackexchange.com/questions/3135606/computing-hyper-area-of-a-contrained-simplex.)










    share|cite|improve this question









    New contributor




    Oscar W. is a new contributor to this site. Take care in asking for clarification, commenting, and answering.
    Check out our Code of Conduct.







    $endgroup$















      6












      6








      6


      1



      $begingroup$


      For any $n geq 2$, let
      $$D_n [r , (a_1, b_1 ) , ldots , (a_n, b_n) ] =
      { (x_1 , ldots , x_n ) in mathbb R^n mid
      sum_i x_i = r mbox{ and } b_i geq x_i geq a_i , forall i },$$

      where $r geq b_i geq a_i geq 0$ for all $i$, and all are real numbers.




      Question: What is the 'volume' of $D_n [r , (a_1, b_1 ) , ldots , (a_n, b_n) ]$?




      So for example for $n=2$, this set is either empty or it is some short line, and the purpose would be to calculate the length of that line (in terms of the parameters $r, a_i, b_i$). This is easy, I've done that already. Also the case $n=3$ would in principle still be doable to do by hand: it would be either zero (in case the set is empty) or part of a plane in $mathbb R^3$, the area of which we desire to compute.



      Now to come up with a formula for the case $n=3$ (and higher) in a smarter way, my idea was to reason inductively from the case $n=2$, so basically reducing the three dimensional case to the two dimensional one, etc. I obviously tried to use integrals.



      The problem I run into, is that in order to compute the volume we want with integrals, we have to view this set as (a subset of) an $n-1$-dimensional space. (Indeed, the integral of the constant function $1$ over the region $D_n [r , (a_1, b_1 ) , ldots , (a_n, b_n) ]$ seen as part of $mathbb R^n$ (rather than $mathbb R^{n-1}$), is equal to $0$. That's not what we want.) But to do that, it seems to me that we would need a concrete isometric embedding of $D_n [r , (a_1, b_1 ) , ldots , (a_n, b_n) ]$ into $mathbb R^{n-1}$, and I can't really find a nice one.



      Do you have an idea about how to approach this problem best?



      (This question was previously posted on Math.StackExchange, see https://math.stackexchange.com/questions/3135606/computing-hyper-area-of-a-contrained-simplex.)










      share|cite|improve this question









      New contributor




      Oscar W. is a new contributor to this site. Take care in asking for clarification, commenting, and answering.
      Check out our Code of Conduct.







      $endgroup$




      For any $n geq 2$, let
      $$D_n [r , (a_1, b_1 ) , ldots , (a_n, b_n) ] =
      { (x_1 , ldots , x_n ) in mathbb R^n mid
      sum_i x_i = r mbox{ and } b_i geq x_i geq a_i , forall i },$$

      where $r geq b_i geq a_i geq 0$ for all $i$, and all are real numbers.




      Question: What is the 'volume' of $D_n [r , (a_1, b_1 ) , ldots , (a_n, b_n) ]$?




      So for example for $n=2$, this set is either empty or it is some short line, and the purpose would be to calculate the length of that line (in terms of the parameters $r, a_i, b_i$). This is easy, I've done that already. Also the case $n=3$ would in principle still be doable to do by hand: it would be either zero (in case the set is empty) or part of a plane in $mathbb R^3$, the area of which we desire to compute.



      Now to come up with a formula for the case $n=3$ (and higher) in a smarter way, my idea was to reason inductively from the case $n=2$, so basically reducing the three dimensional case to the two dimensional one, etc. I obviously tried to use integrals.



      The problem I run into, is that in order to compute the volume we want with integrals, we have to view this set as (a subset of) an $n-1$-dimensional space. (Indeed, the integral of the constant function $1$ over the region $D_n [r , (a_1, b_1 ) , ldots , (a_n, b_n) ]$ seen as part of $mathbb R^n$ (rather than $mathbb R^{n-1}$), is equal to $0$. That's not what we want.) But to do that, it seems to me that we would need a concrete isometric embedding of $D_n [r , (a_1, b_1 ) , ldots , (a_n, b_n) ]$ into $mathbb R^{n-1}$, and I can't really find a nice one.



      Do you have an idea about how to approach this problem best?



      (This question was previously posted on Math.StackExchange, see https://math.stackexchange.com/questions/3135606/computing-hyper-area-of-a-contrained-simplex.)







      co.combinatorics real-analysis integration






      share|cite|improve this question









      New contributor




      Oscar W. is a new contributor to this site. Take care in asking for clarification, commenting, and answering.
      Check out our Code of Conduct.











      share|cite|improve this question









      New contributor




      Oscar W. is a new contributor to this site. Take care in asking for clarification, commenting, and answering.
      Check out our Code of Conduct.









      share|cite|improve this question




      share|cite|improve this question








      edited 14 hours ago









      Iosif Pinelis

      19.5k22259




      19.5k22259






      New contributor




      Oscar W. is a new contributor to this site. Take care in asking for clarification, commenting, and answering.
      Check out our Code of Conduct.









      asked 14 hours ago









      Oscar W.Oscar W.

      311




      311




      New contributor




      Oscar W. is a new contributor to this site. Take care in asking for clarification, commenting, and answering.
      Check out our Code of Conduct.





      New contributor





      Oscar W. is a new contributor to this site. Take care in asking for clarification, commenting, and answering.
      Check out our Code of Conduct.






      Oscar W. is a new contributor to this site. Take care in asking for clarification, commenting, and answering.
      Check out our Code of Conduct.






















          3 Answers
          3






          active

          oldest

          votes


















          5












          $begingroup$

          Computing volumes of polytopes in general is NP-hard. However, your polytope is special - it is a slice of a hypercube by a hyperplane, and this is tractable, see Theorem 1 in:



          Marichal, Jean-Luc; Mossinghoff, Michael J., Slices, slabs, and sections of the unit hypercube, Online J. Anal. Comb. 3, Article 1, 11 p. (2008). ZBL1189.52011.






          share|cite|improve this answer









          $endgroup$





















            2












            $begingroup$

            The $(n-1)$-volume of your polytope (in $mathbb R^n$) equals the $(n-1)$-volume of the polytope
            begin{multline}
            P:={(x_1,dots,x_{n-1})inmathbb R^{n-1}colon \
            a_ile x_ile b_i forall i=1,dots,n-1,\
            a_nle r-sum_1^{n-1}x_ile b_n}
            end{multline}

            (in $mathbb R^{n-1}$) divided by $1/sqrt n$, which latter is the cosine of the angle between the unit vectors $(1/sqrt n,dots,1/sqrt n)$ and $(0,dots,0,1)$ in $mathbb R^n$ -- because $P$ is the image of your polytope under the orthogonal projection of $mathbb R^n$ onto $mathbb R^{n-1}$ given by $(x_1,dots,x_n)mapsto(x_1,dots,x_{n-1})$. The unit vectors $(1/sqrt n,dots,1/sqrt n)$ and $(0,dots,0,1)$ are normal vectors to, respectively, the hyperplane containing your polytope and the hyperplane ${(x_1,dots,x_n)inmathbb R^ncolon x_n=0}$; the latter hyperplane is identified with $mathbb R^{n-1}$.



            A formula for the volume of a polytope was given by Lawrence.






            share|cite|improve this answer











            $endgroup$





















              2












              $begingroup$

              We may assume without loss of generality that $a_i=0$. If
              $r$ and each $b_i$ are positive integers, then consider
              $$ f(x) = frac{left( 1-x^{tb_1+1}right)cdots
              left( 1-x^{tb_n+1}right)}{(1-x)^n}. $$

              The coefficient of $x^{tr}$ is a polynomial function of $t$,
              and the volume $V$ will be its leading coefficient. If I didn't
              make a computational error, then
              $$ V=frac{1}{(n-1)!}sum_{substack{Ssubseteq
              {1,dots,n}\ sum_{iin S}b_i<r}} (-1)^{|S|}
              left(r-sum_{iin S}b_iright)^{n-1}. $$

              If this isn't correct, then something close to it will be.






              share|cite|improve this answer









              $endgroup$













                Your Answer





                StackExchange.ifUsing("editor", function () {
                return StackExchange.using("mathjaxEditing", function () {
                StackExchange.MarkdownEditor.creationCallbacks.add(function (editor, postfix) {
                StackExchange.mathjaxEditing.prepareWmdForMathJax(editor, postfix, [["$", "$"], ["\\(","\\)"]]);
                });
                });
                }, "mathjax-editing");

                StackExchange.ready(function() {
                var channelOptions = {
                tags: "".split(" "),
                id: "504"
                };
                initTagRenderer("".split(" "), "".split(" "), channelOptions);

                StackExchange.using("externalEditor", function() {
                // Have to fire editor after snippets, if snippets enabled
                if (StackExchange.settings.snippets.snippetsEnabled) {
                StackExchange.using("snippets", function() {
                createEditor();
                });
                }
                else {
                createEditor();
                }
                });

                function createEditor() {
                StackExchange.prepareEditor({
                heartbeatType: 'answer',
                autoActivateHeartbeat: false,
                convertImagesToLinks: true,
                noModals: true,
                showLowRepImageUploadWarning: true,
                reputationToPostImages: 10,
                bindNavPrevention: true,
                postfix: "",
                imageUploader: {
                brandingHtml: "Powered by u003ca class="icon-imgur-white" href="https://imgur.com/"u003eu003c/au003e",
                contentPolicyHtml: "User contributions licensed under u003ca href="https://creativecommons.org/licenses/by-sa/3.0/"u003ecc by-sa 3.0 with attribution requiredu003c/au003e u003ca href="https://stackoverflow.com/legal/content-policy"u003e(content policy)u003c/au003e",
                allowUrls: true
                },
                noCode: true, onDemand: true,
                discardSelector: ".discard-answer"
                ,immediatelyShowMarkdownHelp:true
                });


                }
                });






                Oscar W. is a new contributor. Be nice, and check out our Code of Conduct.










                draft saved

                draft discarded


















                StackExchange.ready(
                function () {
                StackExchange.openid.initPostLogin('.new-post-login', 'https%3a%2f%2fmathoverflow.net%2fquestions%2f324878%2fcomputing-the-volume-of-a-simplex-like-object-with-constraints%23new-answer', 'question_page');
                }
                );

                Post as a guest















                Required, but never shown

























                3 Answers
                3






                active

                oldest

                votes








                3 Answers
                3






                active

                oldest

                votes









                active

                oldest

                votes






                active

                oldest

                votes









                5












                $begingroup$

                Computing volumes of polytopes in general is NP-hard. However, your polytope is special - it is a slice of a hypercube by a hyperplane, and this is tractable, see Theorem 1 in:



                Marichal, Jean-Luc; Mossinghoff, Michael J., Slices, slabs, and sections of the unit hypercube, Online J. Anal. Comb. 3, Article 1, 11 p. (2008). ZBL1189.52011.






                share|cite|improve this answer









                $endgroup$


















                  5












                  $begingroup$

                  Computing volumes of polytopes in general is NP-hard. However, your polytope is special - it is a slice of a hypercube by a hyperplane, and this is tractable, see Theorem 1 in:



                  Marichal, Jean-Luc; Mossinghoff, Michael J., Slices, slabs, and sections of the unit hypercube, Online J. Anal. Comb. 3, Article 1, 11 p. (2008). ZBL1189.52011.






                  share|cite|improve this answer









                  $endgroup$
















                    5












                    5








                    5





                    $begingroup$

                    Computing volumes of polytopes in general is NP-hard. However, your polytope is special - it is a slice of a hypercube by a hyperplane, and this is tractable, see Theorem 1 in:



                    Marichal, Jean-Luc; Mossinghoff, Michael J., Slices, slabs, and sections of the unit hypercube, Online J. Anal. Comb. 3, Article 1, 11 p. (2008). ZBL1189.52011.






                    share|cite|improve this answer









                    $endgroup$



                    Computing volumes of polytopes in general is NP-hard. However, your polytope is special - it is a slice of a hypercube by a hyperplane, and this is tractable, see Theorem 1 in:



                    Marichal, Jean-Luc; Mossinghoff, Michael J., Slices, slabs, and sections of the unit hypercube, Online J. Anal. Comb. 3, Article 1, 11 p. (2008). ZBL1189.52011.







                    share|cite|improve this answer












                    share|cite|improve this answer



                    share|cite|improve this answer










                    answered 13 hours ago









                    Igor RivinIgor Rivin

                    79.5k8113309




                    79.5k8113309























                        2












                        $begingroup$

                        The $(n-1)$-volume of your polytope (in $mathbb R^n$) equals the $(n-1)$-volume of the polytope
                        begin{multline}
                        P:={(x_1,dots,x_{n-1})inmathbb R^{n-1}colon \
                        a_ile x_ile b_i forall i=1,dots,n-1,\
                        a_nle r-sum_1^{n-1}x_ile b_n}
                        end{multline}

                        (in $mathbb R^{n-1}$) divided by $1/sqrt n$, which latter is the cosine of the angle between the unit vectors $(1/sqrt n,dots,1/sqrt n)$ and $(0,dots,0,1)$ in $mathbb R^n$ -- because $P$ is the image of your polytope under the orthogonal projection of $mathbb R^n$ onto $mathbb R^{n-1}$ given by $(x_1,dots,x_n)mapsto(x_1,dots,x_{n-1})$. The unit vectors $(1/sqrt n,dots,1/sqrt n)$ and $(0,dots,0,1)$ are normal vectors to, respectively, the hyperplane containing your polytope and the hyperplane ${(x_1,dots,x_n)inmathbb R^ncolon x_n=0}$; the latter hyperplane is identified with $mathbb R^{n-1}$.



                        A formula for the volume of a polytope was given by Lawrence.






                        share|cite|improve this answer











                        $endgroup$


















                          2












                          $begingroup$

                          The $(n-1)$-volume of your polytope (in $mathbb R^n$) equals the $(n-1)$-volume of the polytope
                          begin{multline}
                          P:={(x_1,dots,x_{n-1})inmathbb R^{n-1}colon \
                          a_ile x_ile b_i forall i=1,dots,n-1,\
                          a_nle r-sum_1^{n-1}x_ile b_n}
                          end{multline}

                          (in $mathbb R^{n-1}$) divided by $1/sqrt n$, which latter is the cosine of the angle between the unit vectors $(1/sqrt n,dots,1/sqrt n)$ and $(0,dots,0,1)$ in $mathbb R^n$ -- because $P$ is the image of your polytope under the orthogonal projection of $mathbb R^n$ onto $mathbb R^{n-1}$ given by $(x_1,dots,x_n)mapsto(x_1,dots,x_{n-1})$. The unit vectors $(1/sqrt n,dots,1/sqrt n)$ and $(0,dots,0,1)$ are normal vectors to, respectively, the hyperplane containing your polytope and the hyperplane ${(x_1,dots,x_n)inmathbb R^ncolon x_n=0}$; the latter hyperplane is identified with $mathbb R^{n-1}$.



                          A formula for the volume of a polytope was given by Lawrence.






                          share|cite|improve this answer











                          $endgroup$
















                            2












                            2








                            2





                            $begingroup$

                            The $(n-1)$-volume of your polytope (in $mathbb R^n$) equals the $(n-1)$-volume of the polytope
                            begin{multline}
                            P:={(x_1,dots,x_{n-1})inmathbb R^{n-1}colon \
                            a_ile x_ile b_i forall i=1,dots,n-1,\
                            a_nle r-sum_1^{n-1}x_ile b_n}
                            end{multline}

                            (in $mathbb R^{n-1}$) divided by $1/sqrt n$, which latter is the cosine of the angle between the unit vectors $(1/sqrt n,dots,1/sqrt n)$ and $(0,dots,0,1)$ in $mathbb R^n$ -- because $P$ is the image of your polytope under the orthogonal projection of $mathbb R^n$ onto $mathbb R^{n-1}$ given by $(x_1,dots,x_n)mapsto(x_1,dots,x_{n-1})$. The unit vectors $(1/sqrt n,dots,1/sqrt n)$ and $(0,dots,0,1)$ are normal vectors to, respectively, the hyperplane containing your polytope and the hyperplane ${(x_1,dots,x_n)inmathbb R^ncolon x_n=0}$; the latter hyperplane is identified with $mathbb R^{n-1}$.



                            A formula for the volume of a polytope was given by Lawrence.






                            share|cite|improve this answer











                            $endgroup$



                            The $(n-1)$-volume of your polytope (in $mathbb R^n$) equals the $(n-1)$-volume of the polytope
                            begin{multline}
                            P:={(x_1,dots,x_{n-1})inmathbb R^{n-1}colon \
                            a_ile x_ile b_i forall i=1,dots,n-1,\
                            a_nle r-sum_1^{n-1}x_ile b_n}
                            end{multline}

                            (in $mathbb R^{n-1}$) divided by $1/sqrt n$, which latter is the cosine of the angle between the unit vectors $(1/sqrt n,dots,1/sqrt n)$ and $(0,dots,0,1)$ in $mathbb R^n$ -- because $P$ is the image of your polytope under the orthogonal projection of $mathbb R^n$ onto $mathbb R^{n-1}$ given by $(x_1,dots,x_n)mapsto(x_1,dots,x_{n-1})$. The unit vectors $(1/sqrt n,dots,1/sqrt n)$ and $(0,dots,0,1)$ are normal vectors to, respectively, the hyperplane containing your polytope and the hyperplane ${(x_1,dots,x_n)inmathbb R^ncolon x_n=0}$; the latter hyperplane is identified with $mathbb R^{n-1}$.



                            A formula for the volume of a polytope was given by Lawrence.







                            share|cite|improve this answer














                            share|cite|improve this answer



                            share|cite|improve this answer








                            edited 13 hours ago

























                            answered 13 hours ago









                            Iosif PinelisIosif Pinelis

                            19.5k22259




                            19.5k22259























                                2












                                $begingroup$

                                We may assume without loss of generality that $a_i=0$. If
                                $r$ and each $b_i$ are positive integers, then consider
                                $$ f(x) = frac{left( 1-x^{tb_1+1}right)cdots
                                left( 1-x^{tb_n+1}right)}{(1-x)^n}. $$

                                The coefficient of $x^{tr}$ is a polynomial function of $t$,
                                and the volume $V$ will be its leading coefficient. If I didn't
                                make a computational error, then
                                $$ V=frac{1}{(n-1)!}sum_{substack{Ssubseteq
                                {1,dots,n}\ sum_{iin S}b_i<r}} (-1)^{|S|}
                                left(r-sum_{iin S}b_iright)^{n-1}. $$

                                If this isn't correct, then something close to it will be.






                                share|cite|improve this answer









                                $endgroup$


















                                  2












                                  $begingroup$

                                  We may assume without loss of generality that $a_i=0$. If
                                  $r$ and each $b_i$ are positive integers, then consider
                                  $$ f(x) = frac{left( 1-x^{tb_1+1}right)cdots
                                  left( 1-x^{tb_n+1}right)}{(1-x)^n}. $$

                                  The coefficient of $x^{tr}$ is a polynomial function of $t$,
                                  and the volume $V$ will be its leading coefficient. If I didn't
                                  make a computational error, then
                                  $$ V=frac{1}{(n-1)!}sum_{substack{Ssubseteq
                                  {1,dots,n}\ sum_{iin S}b_i<r}} (-1)^{|S|}
                                  left(r-sum_{iin S}b_iright)^{n-1}. $$

                                  If this isn't correct, then something close to it will be.






                                  share|cite|improve this answer









                                  $endgroup$
















                                    2












                                    2








                                    2





                                    $begingroup$

                                    We may assume without loss of generality that $a_i=0$. If
                                    $r$ and each $b_i$ are positive integers, then consider
                                    $$ f(x) = frac{left( 1-x^{tb_1+1}right)cdots
                                    left( 1-x^{tb_n+1}right)}{(1-x)^n}. $$

                                    The coefficient of $x^{tr}$ is a polynomial function of $t$,
                                    and the volume $V$ will be its leading coefficient. If I didn't
                                    make a computational error, then
                                    $$ V=frac{1}{(n-1)!}sum_{substack{Ssubseteq
                                    {1,dots,n}\ sum_{iin S}b_i<r}} (-1)^{|S|}
                                    left(r-sum_{iin S}b_iright)^{n-1}. $$

                                    If this isn't correct, then something close to it will be.






                                    share|cite|improve this answer









                                    $endgroup$



                                    We may assume without loss of generality that $a_i=0$. If
                                    $r$ and each $b_i$ are positive integers, then consider
                                    $$ f(x) = frac{left( 1-x^{tb_1+1}right)cdots
                                    left( 1-x^{tb_n+1}right)}{(1-x)^n}. $$

                                    The coefficient of $x^{tr}$ is a polynomial function of $t$,
                                    and the volume $V$ will be its leading coefficient. If I didn't
                                    make a computational error, then
                                    $$ V=frac{1}{(n-1)!}sum_{substack{Ssubseteq
                                    {1,dots,n}\ sum_{iin S}b_i<r}} (-1)^{|S|}
                                    left(r-sum_{iin S}b_iright)^{n-1}. $$

                                    If this isn't correct, then something close to it will be.







                                    share|cite|improve this answer












                                    share|cite|improve this answer



                                    share|cite|improve this answer










                                    answered 11 hours ago









                                    Richard StanleyRichard Stanley

                                    28.9k9115189




                                    28.9k9115189






















                                        Oscar W. is a new contributor. Be nice, and check out our Code of Conduct.










                                        draft saved

                                        draft discarded


















                                        Oscar W. is a new contributor. Be nice, and check out our Code of Conduct.













                                        Oscar W. is a new contributor. Be nice, and check out our Code of Conduct.












                                        Oscar W. is a new contributor. Be nice, and check out our Code of Conduct.
















                                        Thanks for contributing an answer to MathOverflow!


                                        • Please be sure to answer the question. Provide details and share your research!

                                        But avoid



                                        • Asking for help, clarification, or responding to other answers.

                                        • Making statements based on opinion; back them up with references or personal experience.


                                        Use MathJax to format equations. MathJax reference.


                                        To learn more, see our tips on writing great answers.




                                        draft saved


                                        draft discarded














                                        StackExchange.ready(
                                        function () {
                                        StackExchange.openid.initPostLogin('.new-post-login', 'https%3a%2f%2fmathoverflow.net%2fquestions%2f324878%2fcomputing-the-volume-of-a-simplex-like-object-with-constraints%23new-answer', 'question_page');
                                        }
                                        );

                                        Post as a guest















                                        Required, but never shown





















































                                        Required, but never shown














                                        Required, but never shown












                                        Required, but never shown







                                        Required, but never shown

































                                        Required, but never shown














                                        Required, but never shown












                                        Required, but never shown







                                        Required, but never shown







                                        Popular posts from this blog

                                        If I really need a card on my start hand, how many mulligans make sense? [duplicate]

                                        Alcedinidae

                                        Can an atomic nucleus contain both particles and antiparticles? [duplicate]